Partial Fraction Integration for (3x+4)/((x^2+4)(3-x))

Click For Summary
The discussion focuses on solving the integral of (3x+4)/((x^2+4)(3-x)) using partial fraction decomposition. The initial approach involved expressing the integrand as (Ax+B)/(x^2+4) + C/(3-x) and multiplying through to find coefficients A, B, and C. After some confusion, the correct values were determined to be A=1, B=0, and C=1. However, the integration step was initially incorrect; the final answer should include a negative sign for the second term, resulting in (1/2)ln(x^2+4) - ln|3-x| + C. The importance of including the modulus sign for the logarithmic term is also emphasized.
tanky322
Messages
43
Reaction score
0

Homework Statement



integral((3X+4)/((x^2+4)(3-x))



Homework Equations





The Attempt at a Solution


I know I should be using partial fractions for this problem but the x^2 in the denom. screws me up. I think the partial fraction should be:

(Ax+B)/(x^2+4) + C/(3-x)

Then:

(Ax+B)(3-x)+C(x^2+4)

Then:

(x^2+4)(3-x)=-Bx-Ax^2+3B+3Ax+Cx^2+C4

So now I am completely lost assuming I am even on the right track.:bugeye:

Thanks Alot!
 
Physics news on Phys.org
(x^2+4)(3-x)=-Bx-Ax^2+3B+3Ax+Cx^2+C4

This makes no sense. Your first steps aren't actual equations, which is confusing you.

(Ax+B)/(x^2+4) + C/(3-x) = (3x+4)/(x^2+4)(3-x)

Multiplying by the two denomiator parts gives

(Ax+B)(3-x) + c(x^2+4) = 3x+4

This is for all x, so plug in clever values (like x=3), to solve for a, b and c
 
I'm not sure what you're doing! You have \frac{3x+4}{(x^2+4)(3-x)}=\frac{Ax+B}{x^2+4}+\frac{C}{3-x}. Now multiply through by the denominator of the LHS to yield 3x+4=(Ax+B)(3-x)+C(x^2+4). Now you need to find three equations to obtain A B and C.
 
Ok it makes more sense now, this partial fraction stuff drives me nuts!


Thanks guys!
 
Ok so I just worked on this problem again... It was driving me nuts! This is what I came out with,
A=1
B=0
C=1

So for a final answer I came up with,
(Ln(x^2+4))/2+Ln(3-x)+ C

Is that right??

Thanks,

Andy
 
tanky322 said:
Ok so I just worked on this problem again... It was driving me nuts! This is what I came out with,
A=1
B=0
C=1

So for a final answer I came up with,
(Ln(x^2+4))/2+Ln(3-x)+ C

Is that right??

Thanks,

Andy

Partial fraction decomposition right. Integration wrong. The second term should have a negative sign. The answer should be \frac{1}{2}\ln{(x^2+4)} - \ln{|3-x|} + C. Don't forget the modulus sign on the second term (it's optional for the first since the square expression is nonnegative for real x).
 
Question: A clock's minute hand has length 4 and its hour hand has length 3. What is the distance between the tips at the moment when it is increasing most rapidly?(Putnam Exam Question) Answer: Making assumption that both the hands moves at constant angular velocities, the answer is ## \sqrt{7} .## But don't you think this assumption is somewhat doubtful and wrong?

Similar threads

  • · Replies 8 ·
Replies
8
Views
2K
  • · Replies 1 ·
Replies
1
Views
1K
Replies
9
Views
2K
Replies
7
Views
2K
Replies
5
Views
2K
  • · Replies 6 ·
Replies
6
Views
2K
  • · Replies 4 ·
Replies
4
Views
2K
Replies
9
Views
2K
  • · Replies 16 ·
Replies
16
Views
2K
Replies
3
Views
2K